Gleim Private Pilot textbook

¡Supera tus tareas y exámenes ahora con Quizwiz!

What is the standard temperature at 20,000 feet?

-25 Celsius

What is the standard temperature at 10,000 feet?

-5 Celsius

Determine the magnetic heading for a flight from Fort Worth Meacham (area 4) to Denton Muni (area 1). The wind is from 330 degrees at 25 knots, the true airspeed is 110 knots, and the magnetic variation is 7 degrees E

003 degrees

Determine the compass heading for a flight from Claxton-Evans County Airport (area 2) to Hampton Varnville Airport (area 1). The wind is from 280 degrees at 8 knots, and the true airspeed is 85 knots. Magnetic variation is 7 degrees W

044 degrees

Determine the magnetic course from Airpark East Airport (area 1) to Winnsboro Airport (area 2) Magnetic variation is 6 degrees 30' E

075 degrees

The number 8 and 26 on the approach ends of the runway indicate that the runway is oriented approximately

080 and 260 magnetic

According to the Chart Supplement, what times can a pilot obtain fuel and services in September at Toledo Express (TOL) Airport?

0800 - 0000 hr. local time

The numbers 9 and 27 on a runway indicated that the runway is oriented approx.

090 and 270 magnetic

Determine the magnetic heading for a flight from Majors Airport (area 1) to Winnsboro Airport (area 2). The wind is from 340 degrees at 12 knots, the true airspeed is 136 knots, and the magnetic variation is 6 degrees 30'E

091 degrees

What are the basic VFR weather minima required to takeoff from the Onawa, IA (K36) airport during the day?

1 statute mile, clear of clouds.

Density Altitude Computations Steps (2)

1) find pressure altitude (altitude when altimeter is 29.92) 2) adjust temperature to convert that pressure altitude to density altitude

Weight/arm/moment calculation computes where the CG is. (3 Steps)

1) multiply weight of each item loaded into the airplane by its arm to determine moment 2) add moments 3) divide total moments by total weight to get CG

If the SSV Class of VORTAC is listed as a Terminal Class, the altitudes and distance to adequately receive the signal of the VORTAC is

1,000 feet to 12,000 feet and 25 NM.

(Refer to Area 3.) The vertical limits of that portion of Class E airspace designated as a Federal Airway over Magee Airport are

1,200 feet AGL to 17,999 feet MSL.

(Refer to Area 1.) What minimum altitude is necessary to vertically clear the obstacle on the northeast side of Airpark East Airport by 500 feet?

1,273 feet MSL.

Refer to Area 3.) What is the floor of the Savannah Class C airspace at the shelf area (outer circle)?

1,300 feet MSL.

(Refer to Area 2.) What minimum altitude is necessary to vertically clear the obstacle on the southeast side of Winnsboro Airport by 500 feet?

1,403 feet MSL.

(Refer to Area 3.) What is the height of the lighted obstacle approximately 6 nautical miles southwest of Savannah International?

1,548 feet MSL.

As standard operating practice, all inbound traffic to an airport without a control tower should continuously monitor the appropriate facility from a distance of

10 miles.

On what course should the VOR receiver (OBS) be set in order to navigate direct from Majors Airport (area 1) to Quitman VORTAC (area 2)

101 degrees

While en route on Victor 185, a flight crosses the 248 degrees radial of Allendale VOR at 0953 and then crosses the 216 degrees radial of Allendale VOR at 1000. What is the estimated time of arrival at Savannah VORTAC

1028.

An aircraft departs an airport in the central standard time zone at 0930 CST for a 2-hour flight to an airport located in the mountain standard time zone. The landing should be at what time

1030 MST

At Sioux Gateway/Col Day (N42°24.16' W96°23.06'), which frequency should be used as a Common Traffic Advisory Frequency (CTAF) to self-announce position and intentions when the control tower is closed?

118.7 MHz.

What is the magnetic heading for a flight from Priest River Airport (area 1) to Shoshone County Airport (area 3)? The wind is from 030 degrees at 12 knots and the true airspeed is 95 knots

121 degrees

(Refer to Area 2 in Figure 22.) What is the correct UNICOM frequency to be used at Coeur D'Alene to request fuel?

122.8 MHz.

Refer to Area 2 in Figure 22.) At Coeur D'Alene, which frequency should be used as a Common Traffic Advisory Frequency (CTAF) to monitor airport traffic?

122.8 MHz.

Refer to Area 2 in Figure 22.) At Coeur D'Alene, which frequency should be used as a Common Traffic Advisory Frequency (CTAF) to self-announce position and intentions?

122.8 MHz.

Refer to Area 5.) What is the CTAF/UNICOM frequency at Barnes County Airport?

122.8 MHz.

Refer to Area 2.) The CTAF/MULTICOM frequency for Garrison Airport is

122.9 MHz.

(Refer to Area 4.) The CTAF/UNICOM frequency at Jamestown Airport is

123.0 MHz.

(Refer to Area 2.) The control tower frequency for Addison Airport is

126.0 MHz.

(Refer to Area 3.) If Dallas Executive Tower is not in operation, which frequency should be used as a Common Traffic Advisory Frequency (CTAF) to monitor airport traffic?

127.25 MHz.

What is the estimated time en route for a flight from Denton (area 1) to Addison (area 2)? The wind is from 200 at 20 knots, the true airspeed is 110 knots, and the magnetic variation is 7 degrees

13 minutes

On a cross-country flight, point A is crossed at 1500 hours and the plan is to reach point B at 1530 hours. Use the following information to determine the indicated airspeed required to reach point B on schedule Distance is 70 NM Forecast wind is 310 degree at 15 knots Pressure altitude 8,000 ft Ambient temperature -10 degrees C True course is 270 degrees The required indicated airspeed would be approximately

137 knots

(Refer to Area 2.) Identify the airspace over Bryn Airport.

14,500 feet MSL.C.Class G airspace -- surface up to but not including 1,200 feet AGL; Class E airspace -- 1,200 feet AGL up to but not including 18,000 feet MSL.

How far will an aircraft travel in 7.5 minutes with a ground speed of 114 knots

14.25 NM

If an aircraft is loaded 90 pounds over maximum certificated gross weight and fuel (gasoline) is drained to bring the aircraft weight within limits, how much fuel should be drained? A.10 gallons B.12 gallons C.15 gallons

15 gallons

Standard temperature and pressure values for sea level

15*C, 29.92" Hg

En route to First Flight Airport (area 5), your flight passes over Hampton Roads Airport (area 2) at 1456 and then over Chesapeake Regional at 1501. At what time should your flight arrive at First Flight

1526

An aircraft departs an airport in the eastern daylight time zone at 0945 EDT for a 2-hour flight to an airport located in the central daylight time zone. The landing should be at what coordinated universal time

1545Z

What are the standard temperature and pressure values for sea level?

15°C and 29.92" Hg.

An aircraft departs an airport in the mountain standard time zone at 1515 MST for a 2-hour 30 minute flight to an airport located in the Pacific standard time zone. What is the estimated time of arrival at the destination airport

1645 PST

An aircraft departs an airport in the central standard time zone at 0845 CST for a 2-hour flight to an airport located in the mountain standard time zone. The landing should be at what coordinated universal time

1645Z

Estimate the time en route from Majors Airport (area 1) to Winnsboro Airport (area 2). The wind is from 340 degrees at 12 knots and the true airspeed is 136 knots. Magnetic variation is 5 degrees east

17 minutes 30 seconds

Determine the magnetic heading for a flight from Sandpoint Airport (area 1 ) to St. Maries Airport (area 4). The wind is from 215 degrees at 25 knots and the true airspeed is 125 knots

172 degrees

An aircraft departs an airport in the mountain standard time zone at 1615 MST for a 2-hour 15 minute flight to an airport located in the Pacific standard time zone. The estimated time of arrival at the destination airport should be

1730 PST

With certain exceptions, Class E airspace extends upward from either 700 feet or 1,200 feet AGL to, but does not include,

18,000 feet MSL.

An aircraft is loaded 110 pounds over maximum certificated gross weight. If fuel (gasoline) is drained to bring the aircraft weight within limits, how much fuel should be drained? A.15.7 gallons B.16.2 gallons C.18.4 gallons

18.4 gallons

(Refer to Area 2.) The elevation of the Chesapeake Regional Airport is

19 feet.

On what course should the VOR receiver (OBS) be set to navigate direct from Hampton Varnville Airport (area 1) to Savannah VORTAC (area 3)

195 degrees

What is the standard temperature at 6,500 feet?

2 Celcius

(Refer to Area 3.) When flying over Arrowwood National Wildlife Refuge, a pilot should fly no lower than

2,000 feet AGL.

Pilots flying over a national wildlife refuge are requested to fly no lower than

2,000 feet AGL.

The terrain elevation of the light tan (light colored) area between Minot (Area 1) and Audubon Lake (Area 2) varies from

2,000 feet to 2,500 feet MSL.

91.169 (IFR flight plan: information required) Intended airports of landing on IFR must have a ceiling and visibility of at least?

2,000 ft and visibility of at least 3SM for one hour before and one hour after ETA or you must have an alternate airport on your flight plan.

Refer to Area 7.) The airspace overlying Mc Kinney (TKI) is controlled from the surface to

2,900 feet MSL.

(Refer to Area 1.) What minimum altitude is required to avoid the Livermore Airport (LVK) Class D airspace?

2,901 feet MSL.

If airborne radar is indicating an extremely intense thunderstorm echo, this thunderstorm should be avoided by a distance of at least

20 miles.

Determine the magnetic course from Cooperstown Airport (area 2) to Jamestown Airport (area 4)

210 degrees

Determine the magnetic heading for a flight from Allendale County airport (area 1) to Claxton-Evans County Airport (area 2). The wind is from 090 degrees at 16 knots and the true airspeed is 90 knots. Magnetic variation is 7 Degrees W

210 degrees

An Aircraft departs an airport in the Pacific standard time Zone at 1030 PST for a 4-hour flight to an airport located in the central standard time zone. The landing should be at what coordinated universal time

2230Z

91.171 (VOR equipment check for IFR operations) The altimeter and transponder have to be checked within (?) months on top of the VOR check.

24 calendar months

If a true heading of 135 degrees results in a ground track of 130 degrees and a true airspeed of 135 knots results in a groundspeed of 140 knots, the wind would be from

246 degrees and 13 knots

Refer to Area 2.) The visibility and cloud clearance requirements to operate VFR during daylight hours over the town of Cooperstown between 1,200 feet AGL and 10,000 feet MSL are

3 miles and 1,000 feet above, 500 feet below, and 2,000 feet horizontally from clouds.

(Refer to Area 1.) The visibility and cloud clearance requirements to operate VFR during daylight hours over Sandpoint Airport at 1,200 feet AGL are

3 miles and 1,000 feet above, 500 feet below, and 2,000 feet horizontally from each cloud.

What is the base of Class B airspace at Lakeview (30F) Airport (Area 2)?

3,000

(Refer to Area 3.) The floor of Class B airspace at Dallas Executive Airport is

3,000 feet MSL.

An aircraft takes off from Gnoss Airport (Area 4) and flies southeast 25.4 NM to Buchanan Airport. What maximum elevation figure would assure obstruction clearance during the flight?

3,200 feet MSL.

(Refer to Area 8.) What minimum altitude is required to fly over the Cedar Hill TV towers in the congested area southwest of Dallas Executive?

3,549 feet MSL.

91.171 (VOR equipment check for IFR operations) Operational checks for VORs must be made every?

30 days

What is the estimated time en route for a flight from Allendale County Airport ( Area 1) to Claxton-Evans County Airport (area 2)? The wind from 100 degree at 18 knots and the true airspeed is 115 knots. Add 2 minutes for climb-out

30 minutes

Determine the estimated time en route for a flight from Priest River Airport (area 1) to Shoshone County Airport (area 3). The wind is from 030 at 12 knots and the true airspeed is 95 knots. Add 2 minutes for climb-out

31 minutes

Determine the magnetic heading for a flight from St. Maries Airport (area 4) is Priest River Airport (area 1). The wind is from 340 degrees at 10 knots and the true airspeed is 90 knots

330 degrees

Determine the magnetic course from First Flight Airport (area 5 ) to Hampton Roads Airport (area 2)

331 degrees

What is the estimated time en route from Sandpoint Airport (area 1) to St. Maries Airport (area 4)? The wind is from 215 degrees at 25 knots, and the true airspeed is 125 knots

34 minutes

Determine the magnetic heading for a flight from Mercer County Reginal Airport (area 3) to Minot International (area 1). the Wind is from 330 degrees at 25 knots, the true airspeed is 100 knots and the magnetic variation is 10 degrees E

352 degrees

What course should be selected on the omnibearing selector (OBS) to make a direct flight from mercer County Regional Airport (area 3) to the Minot VORTAC (area 1) with a TO indication

359 degrees

Refer to Area 3.) Determine the approximate latitude and longitude of Currituck County Airport.

36°24'N - 76°01'W.

What is the estimated time en route for a flight from Claxton-Evans County Airport (area 2) t0 Hampton Varnville Airport (area 1)? The wind is from 290 degrees at 18 knots and the true airspeed is 85 knots. Add 2 minutes for climb-out

39 minutes

91.171 (VOR equipment check for IFR operations) What is the max allowable tolerance during an operational check of a dual VOR system?

4 degree variation between the two indicated bearings

(Refer to Area 4.) The floor of Class B airspace overlying Hicks Airport (T67) north-northwest of Fort Worth Meacham Field is

4,000 feet MSL.

While practicing S-turns, a consistently smaller half-circle is made on one side of the road than on the other, and this turn is not completed before crossing the road or reference line. This would most likely occur in turn

4-5-6 because the bank is increased too rapidly during the early part of the turn

How far will an aircraft travel in 2-1/2 minutes with a groundspeed of 98 knots

4.08 NM

What minimum distance should exist between intense radar echoes before any attempt is made to fly between these thunderstorms?

40 miles.

What is the estimated time en route for a flight from St. Maries Airport (area 4) to Priest River Airport (area 1)? The wind is from 300 degrees at 14 knots and the true airspeed is 90 knots. Add 3 minutes for climb-out

43 minutes

(Refer to Area 3.) The top of the group obstruction approximately 11 nautical miles from the Savannah VORTAC on the 010° radial is

454 feet MSL.

(Refer to Area 2.) What is the approximate latitude and longitude of Cooperstown Airport?

47°25'N - 98°06'W.

(Refer to Area 3.) Determine the approximate latitude and longitude of Shoshone County Airport.

47°33'N - 116°11'W.

What is the estimated time en route from Mercer County Regional Airport (area 3) to Minot International (area 1) ? The wind is from 330 degrees at 25 knots and the true airspeed is 100 knots. Add 3-1/2 minutes for departure and climb out

48 1/2 minutes

The minimum vertical wind shear value critical for probable moderate or greater turbulence is

5 knots per 1,000 feet.

CFR 61.133 (Commercial Pilot Privileges and Limitations) Commercial pilots without an instrument rating cannot carry passengers for hire on cross-country flights during the day beyond?

50NM

What is the approximate base of the cumulus clouds if the temperature at 2,000 feet MSL is 10°C and the dew point is 1°C?

6,000 feet MSL.

FAA advisory circulars containing subject matter specifically related to Airmen are issued under which subject number?

60

FAA advisory circulars containing subject matter specifically related to Airspace are issued under which subject number

70

The controlled airspace located at the Corpus Christi VORTAC (Area 5) begins at

700 feet AGL.

If it is necessary to set the altimeter from 29.15 to 29.85, what change occurs?

700-foot increase in indicated altitude

If the necessary to set the altimeter from 29.15 to 29.82, what change occurs?

700-foot increase in indicated altitude

Estimate the time en route from Addisin (area 2) to Dallas Executive (area 3). The wind is from 300 degrees at 15 knots, the true airspeed is 120 knots, and the magnetic variation is 7 degrees east

8 minutes

AA advisory circulars containing subject matter specifically related to Air Traffic and General Operating Rules are issued under which subject number?

90

CFR 61.23 (Medical Certificates: Requirement and duration) Basicmed allows a pilot to conduct certain VFR and IFR operations using?

A U.S. driver's license instead of a medical certificate as long as they meet all conditions.

CFR 61.113 (Private Pilot Privileges and Limitations) For a pilot acting as PIC and operating under BasicMed what must they have in their logbook?

A completed medical examination checklist and a medical education course completion certificate.

Which type of jetstream can be expected to cause the greater turbulence?

A curving jetstream associated with a deep low-pressure trough.

Which is true with respect to a high- or low-pressure system?

A high-pressure area or ridge is an area of descending air.

In the Northern Hemisphere, the magnetic compass will normally indicate a turn toward the east if

A left turn is entered form a north heading

Which is true regarding high- or low-pressure systems?

A low-pressure area or trough is an area of rising air.

What depicts a Class E airspace that begins at 700 feet AGL?

A magenta vignette that goes around an airport.

What is the difference between area A and area E on the airport depicted?

A may be used for taxi and takeoff; E may be used only as an overrun.

Refer to Area 2.) What kind of airport is Deshler (6D7)?

A public airport with a runway that is not a hard surface.

In the Northern Hemisphere, a magnetic compass will normally indicate initially a turn toward the west if

A right turn is entered from a north heading

What are the characteristics of an unstable atmosphere?

A warm, humid air mass.

(Refer to figure 4) What is the full flap operating range for the airplane? A) 55 to 100 kts B) 55 to 208 kts C) 55 to 165 kts

A) 55 to 100 kts The full flap operating range is indicated by the white arch on the airspeed indicator, the airspeed indicator in fig. 4 indicates the full flap operating range is from 55 to 100 kts

(Refer to figure 3 Altimeter) Altimeter 3 indicates A) 9,500 Feet B) 10,950 Feet C) 15,940 Feet

A) 9,500 Feet Altimeter 3 indicates 9,500 Feet because the shortest needles i near the 1 (i.e. about 10,000 feet). the middle needle is between 9 and the 0. indicating between 9,000 and 10,000 feet. and the long needle is on 5, indicating 500 feet.

(refer to figure 82 altimeter) Altimeter 3 indicates A) 9,500 feet B) 10,500 feet C) 4,500 feet

A) 9,500 feet The long, thin needle is past the fourth tick mark and just befor the 1 (<10,000)

During a spin to the left, which wing(s) is/are stalled? A) Both wings are stalled B) Neither wing is stalled C) Only the left wing is stalled

A) Both wings are stalled

The term "angle of attack" is defined as the angle between the A) Chord line of the wing and the relative wind B) Airplanes longitudinal axis and that of the air striking the airfoil C) Airplanes center line and the relative wind

A) Chord line of the wing and the relative wind

When are the four forces that act on an airplane in equilibrium? A) During unaccelerated level flight B) When the aircraft is accelerating C) When the aircraft is at rest on the ground

A) During unaccelerated level flight

What is one procedure to aid in cooling an engine that is overheating? A) Enrich the fuel mixture. B) Increase the R P M. C) Reduce the airspeed.

A) Enrich the fuel mixture. Enriched fuel mixture have a cooling effect on an engine.

Which is not a primary flight control surface? A) Flaps B) Stabilator C) Ailerons

A) Flaps

Under what condition will true altitude be lower than indicated altitude? A) In colder than standard air temperature. B) In warmer than standard air temperature C) When density altitude is higher than indicated altitude.

A) In colder than standard air temperature. The airplane will be lower than the altimeter indicates when flying in air that is colder than standard temperature. Remember that altimeter readings are adjusted for changes in barometric pressure but not for changes in temperature. When one flies from warmer to colder air and keeps a constant indicated altitude at a constant altimeter setting, the plane has actually descended.

What should be the first action after starting an aircraft engine? A) Just for proper RPM and check for desired indications on the engine gauges. B) Please the magneto or ignition switch momentarily in the off position to check for proper grounding. C) Test each break and the parking brake.

A) Just for proper RPM and check for desired indications on the engine gauges.

Floating caused by the phenomenon of ground effect will be most realized during an approach to land when at A) Less than the length of the wingspan above the surface B) Twice the length of the wingspan above the surface C) A higher than normal angle of attack

A) Less than the length of the wingspan above the surface

What is the relationship of lift, drag, thrust, and weight when the airplane is in straight and level flight? A) Lift equals weight and thrust equals drag B) Lift, drag, and weight equal thrust C) Lift and weight equal thrust and drag

A) Lift equals weight and thrust equals drag

The four forces acting on an airplane in flight are A) Lift, weight, thrust, drag B) Lift, weight, gravity, thrust C) Lift, gravity, power, friction

A) Lift, weight, thrust, drag

During flight, when are the indications of a magnetic compass accurate? A) Only in straight and level unaccelerated flight. B) As long as the airspeed is constant. C) During turns if the bank does not exceed 18"

A) Only in straight and level unaccelerated flight. During flight, the magnetic compass indications can be considered accurate only when in straight and level, unaccelerated flight. During acceleration, deceleration or turns, the compass card will dip and cause false readings

How do variations in temperature affect the altimeter? A) Pressure levels are raised on warm days and the indicated altitude is lower than true altitude. B) Higher temperature expand the pressure levels and the indicated altitude is higher than true altitude. C)Lower temperatures lower the

A) Pressure levels are raised on warm days and the indicated altitude is lower than true altitude. On warm days, the atmospheric pressure levels are higher than on cold days. Your altimeter will indicate a lower than true altitude. remember, "low to high. clear sky"

What action can a pilot take to aid in cooling an engine that is overheating during a climb? A) Reduce rate of climb and increase airspeed. B) Reduce climb speed and increase R P M. C) Increase climb speed and increase R P M.

A) Reduce rate of climb and increase airspeed. If an airplane is overheating during a climb, the engine temperature will be decreased if the airspeed is increased, Airspeed will increase if the rate of climb is reduced

Which device is a secondary flight control? A) Spoilers B) Ailerons C) Stanilators

A) Spoilers

While cruising at 9,500 feet MLS the fuel air mixture is properly adjusted. What we occur if a decent to 4,500 feet MLS is made without adjusting the mixture? A) The Fuel air mixture may become excessively lean. B) There will be more fuel in the cylinders that is needed for normal combustion C) The excessively Rich mixture will create higher cylinder head temperature's and may cause detonation.

A) The Fuel air mixture may become excessively lean.

Which statement is true concerning primary flight controls? A) The effectiveness of each control surface increases with speed because there is more airflow over them B) Only when all three primary flight controls move in sequence do the airflow and pressure distribution change over and around the airfoil C) Primary flight controls include ailerons, rudder, elevator, and trim system

A) The effectiveness of each control surface increases with speed because there is more airflow over them

What force makes an airplane turn? A) The horizontal component of lift B) The vertical component of lift C) Centrifugal force

A) The horizontal component of lift

What determines the longitudinal stability of an airplane? A) The location of the CG with respect to the center of lift B) The effectiveness of the horizontal stabilizer, rudder, and rudder trim tab C) The relationship of trust and lift to weight and drag

A) The location of the CG with respect to the center of lift

What type of fuel can be substituted for an aircraft if the recommended octane is not available? A) The next higher octane aviation gas B) The next lower octane aviation gas C) Unleaded automotive gas of the same octane rating

A) The next higher octane aviation gas

What is ground effect? A) The result of the interference of the surface of the earth with the airflow patterns about an airplane B) The result of an alteration in airflow patterns increasing induced drag about the wings of an airplane C) The result of the disruption of the airflow patterns about the wings of an airplane to the point where the wings will no longer support the airplane in flight

A) The result of the interference of the surface of the earth with the airflow patterns about an airplane

How is engine operation controlled on an engine equipped with a constant speed propeller? A) The throttle controls power output as registered on the manifold pressure gauge and the propeller control regulates engine R P M B) The throttle controls power output as registered on the manifold pressure gauge and the propeller control regulates a constant blade angle C) The throttle controls engine R P M as registered on the tachometer and the mixture control regulates the power output.

A) The throttle controls power output as registered on the manifold pressure gauge and the propeller control regulates engine R P M Airplanes equipped with controllable pitch propellers have both a throttle control and a propeller control. The throttle controls the power output of the engine, which is registered on the manifold pressure gauge. This is a simple barometer that measures the air pressure in the engine intake manifold in inches of mercury. The propeller control regulates the engine R P M, which is registered on a tachometer.

What is true altitude? A) The vertical distance of the aircraft above sea level. B) The vertical distance of the aircraft above the surface. C) The height above the standard datum plane.

A) The vertical distance of the aircraft above sea level. True altitude is the actual altitude above mean sea level, I.E., MSL

Trim systems are designed to do what? A) They relieve the pilot of the need to maintain constant pressure on the flight controls B) They are used during approach and landing to increase wing lift C) They move in the opposite direction from one another to control roll

A) They relieve the pilot of the need to maintain constant pressure on the flight controls

what is the purpose of the rudder on an airplane? A) To control yaw B) To control overbanking tendency C) To control roll

A) To control yaw

Which would most likely cause the cylinder head temperature and engine oil temperature gauges to exceed their normal operating range is? A) Using fuel that has a lower than specified fuel rating B) Using a fuel that has a higher than specified fuel rating C) Operating with higher than normal oil pressure

A) Using fuel that has a lower than specified fuel rating

During the run up at a higher elevation airport a pilot notes a slight engine roughness that is not affected by the magneto check but grows worse during the carburetor heat check under the circumstances what would be the most logical initial action A) check the results obtained with a leaner setting of the mixture B) back to the flight line for a maintenance check C) reduce manifold pressure to control detonation

A) check the results obtained with a leaner setting of the mixture

Generally Speaking, the use of carburetor heat tends to A) decrease engine performance. B) increase engine performance C) have no effect on engine performance.

A) decrease engine performance.

An electrical system failure,, battery and alternator, occurs during flight. In this situation, you should A) experience avionics equipment failure. B) probably experience failure of the engine ignition system, fuel gauges, aircraft lighting system, and avionics equipment. C) probably experience engine failure due to the loss of engine driven fuel pump and also experience failure of the radio and all instruments that require alternating current.

A) experience avionics equipment failure.

The possibility of carburetor icing exists even when the ambient air temperature is as A) high as 70° F and the relative humidity is high. B) high as 95° F and there is visible moisture. C) low as 0° F and the relative humidity is high.

A) high as 70° F and the relative humidity is high.

One purpose of the dual ignition system on an aircraft engine is to provide for A) improved engine performance. B) uniform heat distribution C) balanced cylinder head pressure

A) improved engine performance.

With regard to carburetor ice, float type carburetor systems in comparison to fuel injection systems are generally considered to be A) more susceptible to icing. B) equally susceptible to icing. C) less susceptible to icing.

A) more susceptible to icing.

(Refer to figure 5) A turn coordinator provides an indication of the A) movement of the aircraft about the yaw and roll axes. B) angle of bank up to but not exceeding 30° C) Attitude of the aircraft with reference to the longitudinal axis.

A) movement of the aircraft about the yaw and roll axes. There really are no yaw and roll axes; i.e., an airplane yaws about its vertical axis and rolls about its longitudinal axis. However, this is the best answer since the turn coordinator does indicate the roll and yaw movement of the airplane. The movement of the miniature airplane is proportional to the roll rate of the airplane. When the roll rate is reduced to zero i.e., when the bank is held constant, the instrument provides and indication of the rate of turn.

Detonation me a current high power settings when. A) the fuel mixture ignites instantaneously instead of burning progressively and evenly B) an excessively Rich fuel mixture causes an explosive gain in power. C) The fuel mixture is ignited too early by hot carbon deposits in the cylinder

A) the fuel mixture ignites instantaneously instead of burning progressively and evenly

An abnormally high engine oil temperature indication may be caused by A) the oil level being too low. B) operating with a too high viscosity oil. C) operating with an excessively rich mixture.

A) the oil level being too low. Operating with an excessively low oil level prevents the oil from being cooled adequately; i.e. an inadequate supply of oil will not be able to transfer engine heat to the engine's oil cooler. Insufficient oil may also damage an engine from excessive friction within the cylinders and on other metal to metal contact parts.

What is the effect of advancing the throttle in flight? A. Both aircraft groundspeed and angle of attack will increase B. Airspeed will remain relatively constant but the aircraft will climb C. The aircraft will accelerate, which will cause it to turn to the right

A. Both aircraft groundspeed and angle of attack will increase

Loading an airplane to the most aft CG will cause the airplane to be A. Less stable at all speeds B. Less stable at slow speed, but more stable at high speeds C. Less stable at high speeds, but more stable at low speeds

A. Less stable at all speeds

In what flight condition are torque effects more pronounced in a single engine airplane? A. Low air speed, high power, high angle of attack B. Low air speed, low power, low angle of attack C. High speed, high power, high angle of attack

A. Low air speed, high power, high angle of attack

(Refer to Figure 72 below.) A positive load factor of 2 at 80 mph would cause the airplane to A. Stall B. Break apart C. operate normally, as it is within the normal operating range

A. Stall

During an approach to a stall, an increased load factor will cause the aircraft to A. Stall at a higher airspeed B. Have a tendency to spin C. Be more difficult to control

A. Stall at a higher airspeed

What is the acronym for a computerized command response system that provides automated weather, radio check capability, and airport advisory information selected from an automated menu by microphone clicks?

AUNICOM.

Which feature is associated with the tropopause?

Abrupt change in temperature lapse rate

CFR 61.57 (Pilot in Command) To act as PIC under IFR you must have logged?

Actual or simulated instrument time within the preceding 6 calendar months in the same category of aircraft or in a simulator and must have performed: -at least 6 instrument approaches -holding procedures -intercepting and tracking courses through the use of navigation systems

When converting from true course to magnetic heading, a pilot should

Add westerly variation and subtract left wind correction angle

When taxiing with strong quartering tailwinds, which aileron positions should be used

Aileron down on the side which the wind is blowing

Which aileron position should a pilot generally use when taxiing in strong quartering headwinds

Aileron up on the side from which the wind is blowing

Which condition would cause the altimeter to indicate a lower altitude than true altitude?

Air temperature warmer than standard

Which is true concerning the blue and magenta colors used to depict airports on Sectional Aeronautical Charts?

Airports with control towers underlying Class B, C, D, and E airspace are shown in blue.

Which instrument will become inoperative if the pitot tube becomes clogged?

Airspeed

The pitot system provides impact pressure for which instrument?

Airspeed indicator

Which instrument(s) will become inoperative if the static vents become clogged?

Airspeed, altimeter, and vertical speed

If a pilot changes the altimeter setting from 30.11 to 29.96, what is the approximate change in indication?

Altimeter will indicate 150 feet lower

CFR 61.57 (Pilot in Command) Who can complete a proficiency check?

An FAA inspector, FAA-designated examiner, or a CFII

When operating in Class A airspace what is required?

An IFR clearance.

In the Northern Hemisphere, a magnetic compass will normally indicate a turn toward the north if

An aircraft is accelerated while on an east or west heading

What is a glide path qualification surface? (GQS)

An imaginary surface extending from the runway threshold along the runway centerline to the decision altitude point.

Thunderstorms identified as severe or giving an intense radar echo should be avoided by what distance?

At least 20 miles.

91.169 (IFR flight plan: information required) What are the weather minimums for an airport with an nonprecision approach as your alternate?

At least an 800 ft. ceiling and 2SM visibility at your ETA.

91.167 (Fuel requirements for flight in IFR conditions) An alternate airport is NOT required if the destination airport has?

At least one approved instrument approach procedure and from one hour before to one hour after the ETA a forecast of at least 2,000 ft. ceiling and 3SM of visibility.

Under what condition is pressure altitude and density altitude the same value?

At standard temperature

(Refer to figure 3 Altimeter) Which altimeter(s) indicates (s) more than 10,000 feet? A) 1, 2, and 3 B) 1 and 2 only C) 1 only

B) 1 and 2 only

(refer to figure 82 altimeter) Altimeter 2 indicates A) 500 feet B) 1,500 feet C) 4,500 feet

B) 1,500 feet The long, thin needle is to the right of 0, just before the first tick mark (<2,000) the short needle is halfway between the 1 and 2 (representing 1.5 thousands of feet) and the long, wide needle is on the 5 ( representing 5 hundreds of feet) Therefore the altimeter indicates 1,500 feet.

(Refer to figure 4) What is the maximum flaps extended speed? A) 65 kts B) 100 kts C) 165 kts

B) 100 kts The maximum flaps extended speed is indicated by the upper limit of the white arc. This is the highest airspeed at which a pilot should extend full flaps. At higher airspeed, severe strain or structural failure could result. The upper limit of the white arc on the airspeed indicator shown in fig. 4 indicates 100 kts

(Refer to figure 4) What is the maximum structural cruising speed? A) 100 kts B) 165 kts C) 208 kts

B) 165 kts The maximum structural cruising speed is the maximum speed for normal operation and is indicated as the upper limit of the green arc on an airspeed indicator. The upper limit of the green arc on the airspeed indicator shown in fig 4 indicates 165 kts

(Refer to figure 1 below.) The acute angle A is the angle of A) Incidence B) Attack C) Dihedral

B) Attack

The angle between the chord line of an airfoil and the relative wind is known as the angle of A) Lift B) Attack C) Incidence

B) Attack

Should it become necessary to handprop an airplane engine, it is extremely important that the competent pilot A) Call "contact" before touching propeller. B) Be at the controls in the cockpit. C) Be in the cockpit and call out all commands.

B) Be at the controls in the cockpit.

Ground effect is most likely to result in which problem? A) Settling to the surface abruptly during landing B) Becoming airborne before reaching recommended takeoff speed C) Inability to get airborne even though airspeed is sufficient for normal takeoff needs

B) Becoming airborne before reaching recommended takeoff speed

An aircraft leaving ground effect during takeoff will A) Experience a reduction in ground friction and require a slight power reduction B) Experience an increase in induced drag and a decrease in performance C) Require a lower angle of attack to maintain the same lift coefficient

B) Experience an increase in induced drag and a decrease in performance

Which of the following is true concerning flaps? A) Flaps are attached to the leading edge of the wing and are used to increased wing lift B) Flaps allow an increase in the angle of descent without increasing airspeed C) Flaps are high drag devices deployed from the wings to reduce lift

B) Flaps allow an increase in the angle of descent without increasing airspeed

On aircraft equipped with fuel pumps, when is the auxiliary electric driven pump used? A) All the time to aid the engine driven fuel pump. B) In the event engine driven fuel pump fails. C) Constantly except in starting the engine.

B) In the event engine driven fuel pump fails.

What must a pilot be aware of as a result of ground effect? A) Wingtip vortices increase creating wake turbulence problems for arriving and departing aircraft B) Induced drag decreases; therefore, any excess speed at the point of flare may cause considerable floating C) A full stall landing will require less up elevator deflection than would a full stall when done free of ground effect

B) Induced drag decreases; therefore, any excess speed at the point of flare may cause considerable floating

The elevator controls movement around which axis? A) Longitudinal B) Lateral C) Vertical

B) Lateral

What is an advantage of a constant speed propeller? A) Permits the pilot to select and maintain a desired cruising speed. B) Permits the pilot to select the blade angle for the most efficient performance. C) Provides a smoother operation with stable RPM and eliminates vibrations

B) Permits the pilot to select the blade angle for the most efficient performance.

The uncontrolled firing of the fuel air charge in advance of normal spark ignition is known as A) combustion. B) Pre ignition. C) detonation.

B) Pre ignition.

An airplane said to be inherently stable will A) Be difficult to stall B) Require less effort to control C) Not spin

B) Require less effort to control

What is a benefit of flying with a glass cockpit? A) There is no longer a need to carry paper charts i flight. B) Situational awareness is increased. C) Terrain avoidance is guaranteed.

B) Situational awareness is increased. Glass cockpits are designed to decrease pilot workload, enhanced situational awareness, and increase the safety margin

If the pitot tube and outside static vents become clogged, which instruments would be affected? A) the altimeter, airspeed indicator, and turn and slip indicator. B) The altimeter, airspeed indicator, and vertical speed indicator. C) The altimeter, altitude indicator, and turn and slip indicator

B) The altimeter, airspeed indicator, and vertical speed indicator. The pitot static system is a source of air pressure for the operation of the altimeter, airspeed indicator, and vertical speed indicator. Thus if the pitot and outside static vents become clogged, all these instruments will be affected.

What is pressure altitude? A) The indicated altitude corrected for position and installation error. B) The altitude indicated when the barometric pressure scale is set to 29.92. C) The indicated altitude corrected for nonstandard temperature and pressure.

B) The altitude indicated when the barometric pressure scale is set to 29.92. Pressure altitude is the airplane's height above the standard datum plane of 29.92" Hg. If the altimeter is set to 29.92"Hg. The indicated altitude is the pressure altitude.

What should be the indication on the magnetic compass as you roll into a standard rate turn to the right from a south heading in the northern Hemisphere? A) The compass will initially indicate a turn to the left. B) The compass will indicate a turn to the right, but at a faster rate than is actually occurring. C) The compass will remain on south for a short time, then gradually catch up to the magnetic heading of the airplane

B) The compass will indicate a turn to the right, but at a faster rate than is actually occurring. When on a southerly heading in the Northern Hemisphere and you roll into a standard rate turn to the right, the magnetic compass indication precedes the turn, showing a greater amount of turn then is actually occurring.

What change occurs in the fuel air mixture when carburetor heat is applied? A) A decreased in RPM results from the learn mixture. B) The fuel air mixture becomes richer. C) The fuel air mixture becomes leaner.

B) The fuel air mixture becomes richer.

What is density altitude? A) The height above the standard datum plane. B) The pressure altitude corrected for nonstandard temperature. C)The altitude read directly from the altimeter.

B) The pressure altitude corrected for nonstandard temperature.

Under what condition is indicated altitude the same as true altitude? A) I the altimeter has no mechanical error. B) When at sea level under standard conditions. C)When at 18,00 feet MSL with the altimeter set at 29.92.

B) When at sea level under standard conditions.Indicated altitude ( what your read on your altimeter) approximates the true altitude ( distance above mean sea level) when standard conditions exist and your altimeter is properly calibrated.

Under which condition will pressure altitude be equal to true altitude? A) When the atmospheric pressure is 29.92"Hg. B) When standard atmospheric conditions exist. C) When indicated altitude is equal to the pressure altitude.

B) When standard atmospheric conditions exist. Pressure altitude equals true altitude when standard atmospheric conditions (29.92" Hg and 15°C at sea level) exist.

In the northern Hemisphere, a magnetic compass will normally indicate initially a turn toward the west if A) a left turn is entered form a north heading. B) a right turn is entered from a north heading. C) an aircraft is accelerated while on a north heading.

B) a right turn is entered from a north heading. Due to the northerly turn error in the Northern Hemisphere, a magnetic compass will initially indicate a turn toward the west if a right (east) turn is entered from a north heading.

A precaution for the operation of an engine equipped with a constant speed propeller is to A) avoid high RPM settings with high manifold pressure. B) avoid high manifold pressure settings with low RPM. C) always use a rich mixture with high RPM settings.

B) avoid high manifold pressure settings with low RPM. For any given RPM, there is a manifold pressure that should not be exceeded, Manifold pressure is excessive for a given RPM when the cylinder design pressure is exceeded, placing undue stress on them. If repeated or extended, the stress would weaken the cylinder components and eventually caused engine failure.

Excessively high engine temperatures will A) cause damage to heat conducting hoses and warping of the cylinder cooling fins. B) cause loss of power, excessive oil consumption, and possible permanent internal engine damage. C) not appreciably affect an aircraft engine.

B) cause loss of power, excessive oil consumption, and possible permanent internal engine damage.

Deviation error of the magnetic compass is caused by A) a northerly turning error. B) certain metals and electrical systems within the aircraft. C) the difference in location of true north and magnetic north

B) certain metals and electrical systems within the aircraft. The compass in an airplane will align with any magnetic field, Magnetic fields created by metals and the electrical system of the aircraft will hinder the ability of the compass to align with the earth's magnetic field, This phenomenon is known as deviation. Since deviation error varies by heading, a compass correction card is fitted, providing the pilot with the deviation for a given heading

In the Northern Hemisphere, if an aircraft is accelerated or decelerated, the magnetic compass will normally indicate A) a turn momentarily. B) correctly when on a north or south heading. C) a turn toward the south.

B) correctly when on a north or south heading. Acceleration and deceleration errors on a magnetic compasses do not occur when on a north or south heading in the northern Hemisphere. They occur on east and west headings.

The basic purpose of adjusting the fuel air mixture at altitude is to A) decrease the amount of fuel in the mixture in order to compensate for increased air density B) decrease the fuel flow in order to compensate for decreased air density C) increase the amount of fuel in the mixture to compensate for the decrease in pressure and density of the year Sent from my iPhone

B) decrease the fuel flow in order to compensate for decreased air density

The operating principle of float type carburetors is based on the A) automatic metering of air at the venturi as the aircraft gains altitude. B) difference is air pressure at the venturi throat and the air inlet. C) increase i air velocity in the throat of a venturi causing a increase in air pressure.

B) difference is air pressure at the venturi throat and the air inlet.

Applying carburetor heat will A) result in more air going through the carburetor. B) enrich the fuel air mixture. C) not affect the fuel air mixture.

B) enrich the fuel air mixture.

If a flight is made from an area of high pressure into an area of lower pressure without the altimeter setting being adjusted, the altimeter will indicate A) Lower than the actual altitude above sea level. B) higher than the actual altitude above sea level. C) the actual altitude above sea level.

B) higher than the actual altitude above sea level. When flying from Higher pressure to lower pressure without adjusting your altimeter, the altimeter will indicate a higher than actual altitude, As you adjust an altimeter barometric setting lower, the altimeter indicates lower.

If the pilot suspects that the engine with a fixed pitch propeller is detonating during climb out after takeoff, the initial corrective action to take would be to A) lean the mixture. B) lower than nose slightly to increase the airspeed C) apply carburetor heat.

B) lower than nose slightly to increase the airspeed

If the ignition switch ground wire becomes disconnected, the magneto A) will not operate because the battery is disconnected from the circuit B) may continue to fire. C) will not operate.

B) may continue to fire.

An aircraft which is equipped with an Electronic Flight Display (EFD) can A) compensate for an airman's lack of skill or knowledge B) offer new capabilities and simplify the basic flying task. C) improve flight awareness by allowing the pilot to simply watch for alerts.

B) offer new capabilities and simplify the basic flying task. Electronic Flight Displays offer new capabilities, such as enhanced situational awareness, and simplify basic flying tasks, such as traditional cross country flight planning and fuel management.

A positive indication on the Ammeter A) Indicates the aircraft battery will soon lose its charge. B) shows the rate of charge on the battery. C) means more current is being drawn from the battery then is being replaced.

B) shows the rate of charge on the battery.

What is absolute altitude? A) The altitude read directly from the altimeter. B) The vertical distance of the aircraft above the surface. C)The height above the standard datum plane.

B) the vertical distance of the aircraft above the surface. Absolute altitude is altitude above the surface i.e. AGL

Fuel air ratio is the ratio between the? A) volume of fuel and volume of air entering the cylinder B) weight of fuel and weight of air entering the cylinder C) weight of fuel and weight of air entering the carburetor

B) weight of fuel and weight of air entering the cylinder

(Referred to figure 2 below.) If an airplane weighs 4500 pounds what approximate weight would the airplane structure be required to support during a 45° banked turn while maintaining altitude? A. 4500 pounds B. 6750 pounds C. 7200 pounds

B. 6750 pounds

An airplane has been loaded in such a manner that the CG is located aft of the aft CG limit. One undesirable flight characteristic a pilot might experience with this airplane would be A. A longer takeoff run B. Difficulty in recovering from a stalled condition C. Stalling at higher than normal airspeed

B. Difficulty in recovering from a stalled condition

(Refer to Figure 72 below.) The airspeed indicated by points A and J is A. Maximum structural cruising speed B. Normal stall speed C. Maneuvering speed

B. Normal stall speed

The left turning tendency of an airplane caused by P-factor is the result of A. Clockwise rotation of the engine and the propeller turning the airplane counterclockwise B. Propeller blade descending on the right, producing more thrust then the ascending blade on the left C. Gyroscopic force is applied to the rotating propeller blades acting 90° in advance of the point the force was applied

B. Propeller blade descending on the right, producing more thrust then the ascending blade on the left

The amount of excess load that can be imposed on the wing of an airplane depends upon the A. Position of the CG B. Speed of the airplane C. Abruptness at which the load is applied

B. Speed of the airplane

What causes an airplane (except a T tail) to pitch nose down when power is reduced and controls are not adjusted? A. The CG shifts forward when thrust and drug are reduced B. The downwash on the elevators from the propeller slipstream is reduced and elevator effectiveness is reduced C. when thrust is reduced to less than weight, lift is also reduced and the wings can no longer support the weight

B. The downwash on the elevators from the propeller slipstream is reduced and elevator effectiveness is reduced

Which basic flight maneuver increases the load factor on an airplane as compared to straight and level flight? A. Climbs B. Turns C. Stalls

B. Turns

When does P-factor cause the airplane to yaw to the left? A. When at low angles of attack B. When at high angles of attack C. We're not high airspeeds

B. When at high angles of attack

How barometric pressure, temperature and relative humidity affect density altitude (inversely vs directly)

BP: inversely. BP increase = density altitude decrease Temp: directly. Temp increase = density altitude increase RH: directly. RH increase = density altitude increase

What must a pilot do or be aware of when transitioning an Alert Area?

Be aware that the area may contain unusual aeronautical activity or high volume of pilot training.

(Refer to Area 3.) What is the recommended communications procedure for departure at Currituck County Airport?

Broadcast intentions prior to taxi and announcing runway of departure.

(Refer to Area 2.) What is the recommended communication procedure when inbound to land at Cooperstown Airport?

Broadcast intentions when 10 miles out on the CTAF/MULTICOM frequency, 122.9 MHz.

(Refer to figure 3 Altimeter) Altimeter 1 indicates A) 500 Feet B) 1,500 Feet C) 10,500 Feet

C) 10,500 Feet The altimeter has three needles, The short needle indicates 10,000 foot intervals, the middle length needle indicates 1,000 foot intervals, and the long needle indicates 100 foot intervals, In altimeter 1 The shortest needle is on 1 which indicates about 10,000 feet. The middle length needle indicates half way between zero and 1, which is 500 feet. This is confirmed by the longest needle on 5, indicating 500 feet I.e. 10,500 feet

(Refer to figure 3 Altimeter) Altimeter 2 indicates A) 1,500 Feet B) 4,500 Feet C) 14,500 Feet

C) 14,500 Feet Altimeter 2 indicates 14,500 feet because the shortest needle is between line 1 and the 2 indicating about 15,000 feet. the middle needle is between 4 and 5, indicating 4,500 feet and the long needle is a 5, indicating 500 feet. i.e. 14,500 feet

(Refer to figure 4) What is the caution range of the airplane? A) 0 to 60 kts. B) 100 to 162 kts C) 165 to 208 kts

C) 165 to 208 kts

(Refer to figure 4) The maximum speed at which the airplane can be operated in smooth air is A) 100 kts B) 165 kts C) 208 kts

C) 208 kts The maximum speed at which the airplane can be operated in smooth air is indicated by the red radial line. The airspeed indicator in fig. 4 indicated the red line is at 208 kts

(refer to figure 82 altimeter) Which altimeter(s) indicates (s) more than 9,000 feet? A) 1,2, and 3. B) 1 and 2 only C) 3 only

C) 3 only Altimeter 3 indicates 9,500 feet. altimeter 1 and 2 indicate under 9,000 feet

(refer to figure 82 altimeter) Altimeter 1 indicates A) 4,500 feet B) 1,500 feet C) 500 feet

C) 500 feet The long, thin needle is between 0 and 1 (<10,000) the short needle is between 0 and 1 (<1,000) and the long , wide needle is on 5 (500). Therefor, the altimeter indicates 500 feet.

If it is necessary to set the altimeter from 29.15 to 29.85, what change occurs? A) 70-foot increase in indicated altitude. B) 70-foot increase in density altitude. C) 700-foot increase in indicated altitude.

C) 700-foot increase in indicated altitude. When increasing the altimeter setting form 29.15 to 29.85, the indication altitude increases by 700 feet. The altimeter-indicated altitude moves in the same direction as the altimeter setting and changes about 1,000 feet for every change in 1"Hg in the altimeter setting.

Changes in the center of pressure of a wing affect the aircraft's A) Lift/drag ratio B) Lifting capacity C) Aerodynamic balance and controllability

C) Aerodynamic balance and controllability

Which condition would cause the altimeter to indicate a lower altitude than true altitude? A) Air temperature lower than standard. B) Atmospheric pressure lower than standard. C) Air temperature warmer than standard

C) Air temperature warmer than standard In air that is warmer than standard temperature, the airplane will be higher than the altimeter indicates. Said another way, the altimeter will indicate a lower altitude than actually flown.

Which statement relates to Bernoulli's principle? A) For every action there's an equal and opposite reaction B) An additional upward force is generated as the lower surface of the wing deflects air downward C) Air traveling faster over the curved upper surface of an airfoil causes the lower pressure on the top surface

C) Air traveling faster over the curved upper surface of an airfoil causes the lower pressure on the top surface

which instrument(s) will become inoperative if the static vents become clogged? A) Airspeed indicator only. B) Altimeter only. C) Airspeed indicator, altimeter and vertical speed indicator.

C) Airspeed indicator, altimeter and vertical speed indicator. The pitot static system is a source of air pressure for the operation of the altimeter, airspeed indicator, and vertical speed indicator. Thus if the static vents become clogged, all these instruments will be affected.

Which instrument will become inoperative if the pitot tube becomes clogged? A) Altimeter. B) Vertical speed indicator. C) Airspeed indicator.

C) Airspeed indicator. The pitot static system is a source of pressure for the altimeter, vertical speed indicator. and airspeed indicator. the pitot tube is connected directly to the airspeed indicator and provides impact pressure for i alone, thus, if the pitot tube becomes clogged, only the airspeed indicator will become inoperative.

The pitot system provides impact pressure for which instrument? A) Altimeter. B) Vertical speed indicator. C) Airspeed indicator.

C) Airspeed indicator. The pitot system provides impact pressure , or ram pressure, for only the airspeed indicator

If a pilot changes the altimeter setting from 30.11 to 29.96, what is the approximate change in indication? A) Altimeter will indicate .15"Hg higher. B) Altimeter will indicate 150 feet higher. C) Altimeter will indicate 150 feet lower

C) Altimeter will indicate 150 feet lower Atmospheric pressure decreases approximately 1" of Hg (Mercury) for every 1,000 feet of altitude gained. As an altimeter setting is changed, the change in altitude indication changes the same way (i.e., approximately 1,000 feet for every 1" change in altimeter setting) and in the same direction ( i.e., lowers the altitude reading). Thus, changing from 30.11 to 29.96 is a decreases of .15 in., or 150 feet (.15 x 1,000 feet) lower.

Under what condition is pressure altitude and density altitude the same value? A) At sea level, when the temperature is 0°F. B) When the altimeter has no installation error. C) At standard temperature.

C) At standard temperature. Pressure altitude and density altitude are the same when temperature is standard.

(Refer to figure 7) How should a pilot determine the direction of the bank from a attitude indicator such as the one illustrated? A) By the direction of deflection of the banking scale (A) B) By the direction of the deflection of the horizon bar (B) C) By the relationship of the miniature airplane (C) to the deflected horizon bar (B)

C) By the relationship of the miniature airplane (C) to the deflected horizon bar (B) The direction of the bank on the attitude indicator (A) is indicated by the relationship of the miniature airplane to the deflecting horizon bar. The miniature airplane's relative position to the horizon indicates its attitude; nose high, nose low, left bank, right bank.

If an aircraft is equipped with a fixed pitch propeller and a float type carburetor the first indication of carburetor ice would most likely be A) increased of RPM. B) engine roughness. C) Decrease of RPM.

C) Decrease of RPM.

To properly purge water from the fuel system of an aircraft equipped with fuel tank sumps and a fuel strainer quick drain, it is necessary to drain fuel lines from the A) Fuel strainer drain. B) Lowest point in the fuel system. C) Fuel strainer drain and the fuel tank sumps.

C) Fuel strainer drain and the fuel tank sumps.

During preflight in cold weather, crankcase breather lines should receive special attention because they are susceptible to being clogged by A) congealed oil from the crank case. B) moisture from the outside air which is frozen. C) Ice from the crankcase vapors that have condensed and subsequently frozen.

C) Ice from the crankcase vapors that have condensed and subsequently frozen.

One of the main functions of flaps during approach and landing is to A) Decrease the angle of descent without increasing the airspeed B) Permit a touchdown at a higher indicated airspeed C) Increase the angle of descent without increasing the airspeed

C) Increase the angle of descent without increasing the airspeed

(Refer to figure 4) which color identifies the power off stalling speed in a specified configuration? A) Upper limit of the green arc. B) Upper limit of the white arc. C) Lower limit of the green arc.

C) Lower limit of the green arc. the lower airspeed limit of the green arc indicates the power off stalling speed in a specified configuration. "Specified configuration" Refers to flaps up and landing gear retracted.

(Refer to figure 4) Which color identifies the power off stalling speed with wing flaps and landing gear in the landing configuration? A) Upper limit of the green arc. B) Upper limit of the white arc. C) Lower limit of the white arc.

C) Lower limit of the white arc.

What is an important airspeed limitation that is not color coded on airspeed indicators? A) Never exceed speed. B) Maximum structural cruising speed. C) Maneuvering speed.

C) Maneuvering speed. The maneuvering speed of an airplane is an important airspeed limitation not color coded on the airspeed. It is found in the airplane manual or Placarded in the cockpit. Maneuvering speed is the maximum speed at which full deflection of the airplane controls can be made without incurring structural damage. Maneuvering speed or less should be held in turbulent air to prevent structural damage due to excessive loads.

What does the red line on the airspeed indicator represent? A) Maneuvering speed. B) Turbulent or rough air speed. C) Never exceed speed

C) Never exceed speed The read line on an airspeed indicator indicates the maximum speed at which the airplane can be operated in smooth air, which should never be exceeded intentionally this speed is know as the never exceed speed.

Filling the fuel tanks after the last flight of the day is considered a good operating procedure because it will A) Force any existing water to the top of the tank away from the fuel lines to the engine B) Prevent expansion of fuel by a limiting airspace in the tanks C) Pregnant moisture condensation by illuminating airspace in the tanks

C) Pregnant moisture condensation by illuminating airspace in the tanks

What steps must be taken when flying with glass cockpits to ensure safe flight? A) Use the moving map for primary means of navigation, use the multi functional display to check engine systems and weather, back up with supplementary forms of information B) Regularly scan each item on the primary flight display, confirm on the multi functional display. C) Regularly scan both inside and outside, use all appropriate checklists, and cross check with other forms of information

C) Regularly scan both inside and outside, use all appropriate checklists, and cross check with other forms of information A regular scan , both visually outside and inside on backup gauges, should be combined with other means of navigation and checklist to endure safe flight.

As altitude increases, the indicated airspeed at which a given airplane stalls in a particular configuration will A) Decrease as the true airspeed decreases B) Decrease as the true air speed increases C) Remain the same regardless of altitude

C) Remain the same regardless of altitude

The angle of attack at which an airplane wing stalls will A) Increase if the CG is moved forward B) Change with an increase in gross weight C) Remain the same regardless of gross weight

C) Remain the same regardless of gross weight

In what flight condition must an aircraft be placed in order to spin? A) Partially stalled with one wing low B) In a steep diving spiral C) Stalled

C) Stalled

Which condition is most favorable to have development of carburetor icing? A) Any temperature below freezing and a relative humidity of less than 50 percent. B) Temperature between 32°F and 50°F and low humidity C) Temperature between 20°F and 70°F and high humidity

C) Temperature between 20°F and 70°F and high humidity

Which of the following is a true statement concerning electrical systems? A) The master switch provide current to the electrical system B) The airspeed indicator is driven by the electrical system C) The lights and radio use the electrical system for power

C) The lights and radio use the electrical system for power

(Refer to figure 4) Which marking identifies the never exceed speed? A) Upper limit of the green arc. B) Upper limit of the white arc. C) The red radial line.

C) The red radial line. The red radial line represents the never exceed speed (Vne). Operating an aircraft beyond Vne may result in severe damage

detonation occurs in reciprocating aircraft engine when A) The spark plug is fouled or shortened out or the wiring is defective. B) Hot spots in the combustion chamber ignite the fuel air mixture in advance of normal ignition. C) The unburned charge in the cylinders explode instead of burning normally

C) The unburned charge in the cylinders explode instead of burning normally

(Refer to figure 4) Which color identifies the normal flap operating range? A) The yellow arc. B) The green arc. C) The white arc.

C) The white arc.

The presence of carburetor ice in an aircraft equipped with a fixed pitch propeller can be verified by applying carburetor heat and noting A) an increase in RPM and then a gradual decrease in RPM B) a decrease in RPM and then a constant RPM indication C) a decrease in RPM and then a gradual increase in RPM

C) a decrease in RPM and then a gradual increase i RPM

In the Northern Hemisphere, a magnetic compass will normally indicate initially a turn toward the east if A) an aircraft is decelerated while on a south heading. B) an aircraft is accelerated while on a north heading C) a left turn is entered from a north heading.

C) a left turn is entered from a north heading. In the Northern Hemisphere, a magnetic compass normally initially indicates a turn toward the east if a left (west) turn is entered from a north heading.

In the Northern Hemisphere, a magnetic compass will normally indicate a turn toward the north if A) an aircraft is decelerated while on an east or west heading B) a left turn is entered from a west heading. C) an aircraft is accelerated while on an east or west heading.

C) an aircraft is accelerated while on an east or west heading In the Northern Hemisphere, a magnetic compass will normally indicate a turn toward the north if an airplane is accelerated while on an east or west heading.

Excessively high engine temperatures, either in the air or on the ground, will A) increase fuel consumption and may increase power due to the increased heat. B) result in damage to heat conducting hose and warping of cylinder cooling fans. C) cause loss of power, excessive oil consumption, and possible permanent internal engine damage.

C) cause loss of power, excessive oil consumption, and possible permanent internal engine damage.

If the grade of fuel used in an aircraft engine is lower than specified for the engine, it will most likely causes A) a mixture of fuel and air that is not uniform in all cylinders. B) lower cylinder head temperatures. C) detonation.

C) detonation.

To keep the battery charged, the alternator voltage output should be A) less than the battery voltage B) equal to the battery voltage C) higher than the battery voltage

C) higher than the battery voltage

If an aircraft is equipped with a fixed pitch propeller and a float type carburetor the first indication of carburetor ice would most likely be A) a drop in oil temperature and cylinder head temperature. B) engine roughness. C) loss of RPM

C) loss of RPM

If a flight is made from an area of low pressure into and area of high pressure without the altimeter setting being adjusted, the altimeter will indicate A) the actual altitude above seal level. B) higher than the actual altitude above sea level. C) lower than the actual altitude above sea level.

C) lower than the actual altitude above sea level. When an altimeter setting is at a lower values than the correct setting, the altimeter is indicating less than i should and thus would be showing lower than the actual altitude above sea level

Deviation in a magnetic compass is caused by the A) presence of flaws in the permanent magnets of the compass. B) difference in the location between true north and magnetic north C) magnetic fields within the aircraft distorting the lines of magnetic force.

C) magnetic fields within the aircraft distorting the lines of magnetic force. Magnetic fields produced by metals and electrical accessories in the airplane disturb the compass needle and produce errors, These errors are referred to as compass deviation.

(Refer to figure 7) The proper adjustment to make on the attitude indicator during level flight is to align the A) horizon bar to the level flight indication. B) horizon bar to the miniature airplane. C) miniature airplane to the horizon bar.

C) miniature airplane to the horizon bar. The horizon bar, marked as B on figure 7 represents the true horizon, This bar is fixed to the gyro and remains on a horizontal plane as the airplane is pitched or banked about its lateral or longitudinal axis, indicating the attitude of the airplane relative to the true horizon. An adjustment knob is provided, with which the pilot may move the miniature airplane marked as C up or down to align the miniature airplane with the horizontal bar to suit the pilot's line of vision.

(Refer to figure 6) To receive accurate indications during flight from a heading indicator, the instruments must be A) set prior to flight on a known heading. B) calibrated on a compass rose at regular intervals. C) periodically realigned with the magnetic compass as the gyro precesses.

C) periodically realigned with the magnetic compass as the gyro precesses. Due to gyroscopic precession, directional gyros must be periodically realigned with a magnetic compass

In the Northern Hemisphere, the magnetic compass will normally indicate a turn toward the south when A) a left turn is entered from an east heading. B) a right turn is entered from a west heading. C) the aircraft is decelerated while on a west heading.

C) the aircraft is decelerated while on a west heading. In the Northern Hemisphere, a magnetic compass will normally indicate a turn toward the south if an airplane is decelerated while on a east or west heading.

For internal cooling, air cooled engines are especially dependent on A) a properly functioning thermostat. B) air flowing over the exhaust manifold. C) the circulation of lubricating oil.

C) the circulation of lubricating oil. An engine accomplishes much of its cooling by the flow of oil through the lubrication system, The lubrication system aids in cooling by reducing friction and absorbing heat from internal engine parts. Many airplane engines use an oil cooler, a small radiator device that will cool the oil before it is recirculated through the engine.

If the engine oil temperature and cylinder head temperature gauges have exceeded their normal operating range, the pilot may have been operating with A) the mixture set too rich. B) higher than normal oil pressure. C) too much power and with the mixture set too lean.

C) too much power and with the mixture set too lean. If the engine oil temperature and cylinder head temperature gauge exceed their normal operating range, it is possible that the power setting is too high and the fuel air mixture is set excessively lean, These conditions may cause engine overheating.

Altimeter setting is the value to which the barometric pressure scale of the altimeter is set so the altimeter indicates A) calibrated altitude at field elevation. B) absolute altitude at field elevation. C) true altitude at field elevation.

C) true altitude at field elevation. Altimeter setting is the value to which the scale of the pressure altimeter is set so that the altimeter indicates true altitude at field elevation.

(Refer to Figure 72 below.) What load factor would be created if positive 15 feet per second gusts were encountered at 120 mph. A. 2.8 B. 3.0 C. 2.0

C. 2.0

(Refer to figure 2 below.) If an airplane weighs 3300 pounds what approximate weight would the airplane structure be required to support during a 30° banked turn while maintaining altitude? A. 1200 pounds B. 3100 pounds C. 3960 pounds

C. 3960 pounds

(Refer to figure 2 below.) If an airplane weighs 2300 pounds, what approximate weight with the airplane structure be required to support during a 60° banked turn while maintaining altitude? A. 2300 pounds B. 3400 pounds C. 4600 pounds

C. 4600 pounds

An airplane loaded with the center of gravity (CG) rear of the aft CGA limit could A. Make it easier to recover from stalls and spins B. Make it more difficult to flare for landing C. Increase the likelihood of inadvertent overstress

C. Increase the likelihood of inadvertent overstress

Limit load factor is the ratio of A. Angle of attack to stall speed B. Angle of attack to power on configuration-specific stall speed C. Maximum sustainable load to the gross weight of the airplane

C. Maximum sustainable load to the gross weight of the airplane

Structural damage or failure is more likely to occur in smooth air at speed above A. Vno B. Va C. Vne

C. Vne

CFR 61.133 (Commercial Pilot Privileges and Limitations) Commercial pilots without an IFR rating can't?

Carry passengers at night.

Deviation in a magnetic compass is caused by the

Certain metals and electrical systems within the aircraft.

CFR 91.21 (Portable electronic devices) What is prohibited on aircraft that are being operated under IFR?

Certain portable electronic devices

Which sources of aeronautical information, when used collectively, provide the latest status of airport conditions (e.g., runway closures, runway lighting, snow conditions)?

Chart Supplement and Distant (D) NOTAMs.

Information concerning parachute jumping sites may be found in the

Chart Supplement.

(Refer to Area 4.) The airspace directly overlying Fort Worth Meacham is

Class D airspace to 3,200 feet MSL.

A blue segmented circle on a Sectional Chart depicts which class airspace?

Class D.

Refer to Area 6.) The NALF Fentress (NFE) Airport is in what type of airspace?

Class E.

(Refer to Area 3.) Identify the airspace over Sprague Airport.

Class G airspace -- surface up to but not including 1,200 feet AGL; Class E airspace - 1,200 feet AGL up to but not including 18,000 feet MSL.

(Refer to east of Area 5.) The airspace overlying and within 5 miles of Barnes County Airport is

Class G airspace from the surface to 700 feet AGL.

Which conditions are favorable for the formation of a surface based temperature inversion?

Clear, cool nights with calm or light wind.

According to the Chart Supplement, what are the operational requirements of a VORTAC?

Collocated VOR and TACAN navigational facilities.

Which is a characteristic typical of a stable air mass?

Continuous precipitation.

What feature is normally associated with the cumulus stage of a thunderstorm?

Continuous updraft.

Which is true regarding the development of convective circulation?

Cool air must sink to force the warm air upward.

Why does the wind have a tendency to flow parallel to the isobars above the friction level?

Coriolis force tends to counterbalance the horizontal pressure gradient.

What prevents air from flowing directly from high-pressure areas to low-pressure areas?

Coriolis force.

In flying the rectangular course, when should the aircraft bank vary from a steep bank to a medium bank

Corner 3

In flying the rectangular course, when would the aircraft be turned less than 90 degrees

Corners 1 and 4

In the Northern Hemisphere, if an aircraft is accelerated or decelerated, the magnetic compass will normally indicate

Correctly when on a north or south heading

A moist, unstable air mass is characterized by

Cumuliform clouds and showery precipitation.

Which are characteristics of a cold air mass moving over a warm surface?

Cumuliform clouds, turbulence, and GOOD visibility.

What visible signs indicate extreme turbulence in thunderstorms?

Cumulonimbus clouds, very frequent lightning, and roll clouds.

The airspace surrounding the Gila Bend AF AUX Airport (GXF) (Area 6) is classified as Class

D.

At which airports is fixed-wing Special VFR not authorized?

Dallas-Fort Worth International and Dallas Love Field.

During the life cycle of a thunderstorm, which stage is characterized predominately by downdrafts?

Dissipating.

(Refer to Area 3.) What is the airspace classification around Findlay (FDY) airport?

E.

When are two-way radio communications required on a flight from Bishop Airport (Area 4) to McCampbell Airport (Area 1) at an altitude of 2,000 feet MSL?

Entering the Corpus Christi Class C airspace.

Moisture is added to a parcel of air by

Evaporation and sublimation

What action should a pilot take when operating under VFR in a Military Operations Area (MOA)?

Exercise extreme caution when military activity is being conducted.

Forward vs aft center of gravity (in regards to downward force on tail and lift)

F: greater downward force on tail, total lift required is increased A: less downward force on the tail, less lift required

Refer to Area 2.) Which airport is located at

Fischer.

91.169 (IFR flight plan: information required) If no instrument approaches are prescribed, the minimums for listing as airport as your alternate on an IFR flight are?

Forecast weather allowing descent from the MEA approach, and landing under basic VFR.

What is indicated if ice pellets are encountered at 8,000 feet?

Freezing rain at higher altitude.

91.171 (VOR equipment check for IFR operations) What is the max allowable variation for a ground-based VOT? Airborne checkpoint?

Ground based VOT- +/- 4 degrees, Airborne checkpoint- +/- 6 degrees

The line from point C to point B of the wind triangle represents

Groundspeed and true course

Headwind and tailwind vs landing distance (and why)

H: reduce landing distance because the ground speed at touchdown is reduced T: increase landing distance because the ground speed at touchdown is increased

Headwind and tailwind vs takeoff run and angle of climb

H: shorten takeoff run, increase angle of climb T: increase takeoff run, decrease angle of climb

Which statement is true concerning the hazards of hail?

Hailstones may be encountered in clear air several miles from a thunderstorm.

91.109 (Flight instruction; simulated instrument flight and certain flight tests) To operate an airplane in simulated instrument flight you must?

Have at least a private pilot who is appropriately rated in your aircraft occupying the other control seat as a safety pilot.

CFR 61.23 (Medical Certificates: Requirement and duration) What are the conditions a pilot is allowed to conduct certain VFR and IFR operates using a U.S. driver's license under?

Have held an FAA medical certificate at any time after July 14, 2006 Completes an approved medical education course in the last 24 calendar months in accordance with 14 CFR part 68 Receives a comprehensive medical examination from a state-licensed physician in the previous 48 months in accordance with 14 CFR part 68

If a flight is made form an area of high pressure into an area of lower pressure without the altimeter setting being adjusted, the altimeter will indicate

Higher than the actual altitude above sea level

If a flight is made from an area of high pressure into an area of lower pressure without the altimeter setting being adjusted, the altimeter will indicate

Higher than the actual altitude above sea level

(Refer to Area 3.) What type military flight operations should a pilot expect along IR 644?

IFR training flights above 1,500 feet AGL at speeds in excess of 250 knots.

91.123 (Compliance with ATC clearances and instructions) During an IFR to IMC flight, if a distress condition is encountered the pilot should?

Immediately declare an emergency and obtain an amended clearance.

The most important rule to remember in the event of a power failure after becoming airborne is to

Immediately establish the proper gliding altitude and airspeed

Under what condition will true altitude be lower than indicated altitude?

In colder than standard air temperature

You are preflight planning in the morning before an afternoon flight. Where would you find information regarding an "Airport surface hot spot?"

In the Chart Supplements U.S.

91.169 (IFR flight plan: information required) What are the weather minimums for an airport with an precision approach as your alternate?

Indicate at least 600 ft. ceiling and 2SM visibility at your ETA.

If more than one cruising altitude is intended, which should be entered in item 15, "Level," of the flight plan

Initial cruising altitude

With respect to advection fog, which statement is true?

It can appear suddenly during day or night, and it is more persistent than radiation fog.

Which sign is a designation and direction of an exit taxiway from a runway?

K

(Refer to area C.) Hour should the flight controls be held while taxiing a tricycle-gear equipped airplane with a left quartering tailwind

Left aileron down, elevator down

(Refer to area C.) How should the flight controls be held while taxiing a tailwind airplane with a left quartering tailwind

Left aileron down, elevator down

(Refer to area A.) How should the flight controls be held while taxing a tricycle-gear equipped airplane into a left quartering headwind

Left airleron up elevator neutral

Upon encountering severe turbulence, which flight condition should the pilot attempt to maintain

Level flight altitude

When approaching Lincoln Municipal from the west at noon for the purpose of landing, initial communications should be with

Lincoln Approach Control on 124.0 MHz.

Which statement about longitude and latitude is true?

Lines of longitude cross the Equator at right angles.

To minimize the side loads placed on the landing gear during touchdown, the pilot should keep the

Longitudinal axis of the aircraft parallel to the direction of its motion

Which color identifies the power-off stalling speed in a specified configuration?

Lower limit of the green arc

Which color identifies the power-off stalling speed with wing flaps and landing gear in the landing configuration?

Lower limit of the white arc

If a flight is made form an area of low pressure into an area of high pressure without the altimeter setting being adjusted, the altimeter will indicate

Lower than the actual altitude above sea level

If a flight is made from an area of low pressure into an area of high pressure without the altimeter setting being adjusted, the altimeter will indicate

Lower than the actual altitude above sea level

Deviation in magnetic compass is caused by the

Magnetic fields within the aircraft distorting the lines of magnetic force

The angular difference between true north magnetic north is

Magnetic variation

(Refer to Area 2.) Which airport is located at approximately 47°34'30"N latitude and 100°43'00"W longitude?

Makeeff.

What is an important airspeed limitation that is not color coded on airspeed indicators?

Maneuvering speed

(Refer to Area 2.) What hazards to aircraft may exist in areas such as Devils Lake East MOA?

Military training activities that necessitate acrobatic or abrupt flight maneuvers.

Which public use airport depicted is indicated as having fuel?

Minot Int'l (Area 1).

(Refer to Area 1.) What minimum radio equipment is required to land and take off at Norfolk International?

Mode C transponder and two-way radio.

What is the recommended communications procedure for landing at Lincoln Municipal during the hours when the tower is not in operation?

Monitor airport traffic and announce your position and intentions on 118.5 MHz.

91.177 (Minimum altitudes for IFR operations) Except when necessary for takeoff or landing, the minimum altitude for IFR is (?) for mountainous and nonmountainous terrain.

Mountainous- 2,000 ft. above highest obstacle within horizontal distance of 4NM over designated mountainous terrain Nonmountainous- 1,000 ft. above highest obstacle within horizontal distance of 4NM over nonmountainous terrain

91.167 (Fuel requirements for flight in IFR conditions) What is the fuel requirements for flight in IFR conditions?

Must carry sufficient fuel to fly to first airport of intended landing, fly to the alternate airport if required, and then fly 45 minutes at normal cruising speed.

When information is disseminated about a taxiway closure, it will be located in

NOTAM (D) distribution.

What does the red line on an airspeed indicator represent?

Never-exceed speed

CFR 61.113 (Private Pilot Privileges and Limitations) What are the limitations of flight for a PIC who is operating under BasicMed?

No portion of the flight may be conducted above 18,000 ft. MSL, be outside of the U.S. unless authorized by country which flight is being conducted, and cannot have an indicated airspeed greater than 250 kts.

Where is Loup City Municipal located with relation to the city?

Northwest approximately 1 mile.

91.123 (Compliance with ATC clearances and instructions) What must you do if you deviate from an ATC clearance in an emergency?

Notify ATC asap

When executing an emergency approach to land in a single-engine airplane, it is important to maintain a constant glide speed because variations in glide speed will

Nullify all attemps at accuracy in judgment of gliing distance and landing spot

CFR 91.103 (Preflight Action) Before any IFR flight the pilot must?

Obtain and be familiar with information about weather reports and forecasts, fuel requirements, alternatives available if planned flight can't be completed, known traffic delays, runway lengths, and takeoff and landing distances.

During flight, when are the indications of a magnetic compass accurate?

Only in straight-and-level unaccelerated flight

Of the following, which is accurate regarding turbulence associated with thunderstorms?

Outside the cloud, shear turbulence can be encountered 20 miles laterally from a severe storm.

Who is primarily responsible for maintaining an aircraft in airworthy condition

Owner or operator

CFR 61.57 (Pilot in Command) If a pilot fails to meet the experience requirements during the prescribed time or six calendar months after they must?

Pass a proficiency check.

What are the characteristics of stable air?

Poor visibility; steady precipitation; stratus clouds.

Which in-flight hazard is most commonly associated with warm fronts?

Precipitation-induced fog.

What causes wind?

Pressure differences.

How do variations in temperature affect the altimeter?

Pressure levels are raised on warm days and the indicated altitude is lower than true altitude

Which wind condition would be most critical when taxiing a nosewheel equipped high-wing airplane

Quartering tailwind

In what ways do advection fog, radiation fog, and steam fog differ in their formation or location?

Radiation fog is restricted to land areas; advection fog is most common along coastal areas; steam fog forms over a water surface.

What steps must be taken when flying with glass cockpits to ensure safe flight?

Regularly scan both inside and outside, use all appropriate checklist, and cross-check with other forms of information

91.123 (Compliance with ATC clearances and instructions) If you are given priority by ATC in an emergency, ATC can?

Request a detailed report within 48 hours to the manager of that facility.

(Refer to area B.) How should the flight controls be held while taxiing a tailwheel airplane into a right quartering headwind

Right aileron up, elevator up

Refer to Crawford Airport (N38°42.25' W107°38.62'). What is the traffic pattern for Runway 25?

Right hand traffic pattern.

Which public use airports depicted are indicated as having fuel?

Rockwall (Area 1) and Sulphur Springs (Area 5).

Which type radar service is provided to VFR aircraft at Lincoln Municipal?

Sequencing to the primary Class C airport, traffic advisories, conflict resolution, and safety alerts.

What is a benefit of flying with a glass cockpit?

Situational awareness is increased

When an air mass is stable, which of these conditions is most likely to exist?

Smoke, dust, haze, etc., concentrated at the lower levels with resulting poor visibility.

Select the four flight fundamentals involved in maneuvering an aircraft

Straight-and-level flight, turns, climbs, and descents

What is a characteristic of stable air?

Stratiform clouds

What type weather can one expect from moist, unstable air, and very warm surface temperature?

Strong updrafts and cumulonimbus clouds.

When conditionally unstable air with high-moisture content and very warm surface temperature is forecast, one can expect what type of weather?

Strong updrafts and cumulonimbus clouds.

Bests speeds for takeoff, rate-of-climb and glide

T: 65 mph RofC: 85 mph G: 75 mph

Tailwind vs headwind

T: winds that push from behind the plane H: winds that push from the front of the plane

According to the airport diagram, which statement is true?

Takeoffs may be started at Position A on Runway 12, and the landing portion of this runway begins at position B

That portion of the runway identified by the letter A may be used for

Taxiing and takeoff

What information should be entered in item 16, "Destination Aerodrome," for a VFR day Flight

The ICAO four-letter indicator of destination airport if no stopover for more than 1 hour is anticipated

CFR 91.3 (Responsibility and Authority of the PIC) Who has final authority and is responsible to determining the airworthiness and operation of that aircraft prior to each flight?

The PIC

In the Northern Hemisphere, a magnetic compass will normally indicate initially a turn toward the south when

The aircraft is decelerated while on a west heading

CFR 61.113 (Private Pilot Privileges and Limitations) What are the limitations of an aircraft for a PIC who is operating under BasicMed?

The aircraft may not be certified to carry more than six occupants and cannot have a maximum takeoff weight that is greater than 6,000lbs.

If the pitot tube and outside static vents become clogged, which instruments would be affected?

The altimeter, airspeed indicator, and vertical speed indicator

What is pressure altitude?

The altitude indicated when the barometric pressure scale is set to 29.92

From which measurement of the atmosphere can stability be determined?

The ambient lapse rate.

What information should be entered in item 19, "Endurance,: for a VFR day flight

The amount of usable fuel on board expressed in time

What information should be entered in item 15, "Level" for a VFR day flight

The appropriate VFR cruising altitude

Select the true statement pertaining to the life cycle of a thunderstorm.

The beginning of rain at the Earth's surface indicates the mature stage of the thunderstorm.

What should be the indication on the magnetic compass as you roll into a standard rate turn to the right from a south heading in the Northern Hemisphere?

The compass will indicate a turn to the right, but at a faster rate than is actually occurring

91.171 (VOR equipment check for IFR operations) When making VOR operation checks what should be listed in the aircraft log?

The date, place, bearing error, and pilot signature.

What information should be entered into item 16, "Destination Aerodrome,: for a VFR day flight

The destination airport identifier code and city name

CFR 61.51 (Pilot logbooks) What must be logged in the logbook when adding instrument flight time?

The location and type of each instrument approach completed and the name of the safety pilot.

91.169 (IFR flight plan: information required) When a pilot is proceeding to an alternate airport the landing minimums used should be?

The minimums specified for the approach procedure selected.

During the preflight inspection who is responsible for determining the aircraft as safe for flight

The pilot in command

How should a VFR flight plan be closed at the completion of the flight at a controlled airport

The pilot must close the flight plan with the nearest FSS or other FAA facility upon landing

What is density altitude?

The pressure altitude corrected for nonstandard temperature

Which is true regarding the use of airborne weather-avoidance radar for the recognition of certain weather conditions?

The radar scope provides no assurance of avoiding instrument weather conditions.

Which marking identifies the never-exceed speed?

The red radical line

VFR approaches to land at night should be accomplished

The same as during daytime

What determines the structure or type of clouds which will form as a result of air being forced to ascend?

The stability of the air before lifting occurs.

Which weather phenomenon signals the beginning of the mature stage of a thunderstorm?

The start of rain.

What information should be entered in item 19, "Endurance," for a VFR day flight

The total amount of usable fuel onboard expressed in hours and minutes

What is true altitude?

The vertical distance of the aircraft above sea level

What is absolute altitude?

The vertical distance of the aircraft above the surface

Which color identifies the normal flap operating range?

The white arc

Which statement is true regarding squall lines?

They are nonfrontal and often contain severe steady-state thunderstorms

91.173 (ATC clearance and flight plan required) No person may operate an aircraft in controlled airspace under IFR unless?

They filed an IFR flight plan or received an ATC clearance.

Which statement is true concerning squall lines?

They offer the most intense weather hazards to aircraft

How should an aircraft preflight inspection be accomplished for the first flight of the day

Thorough and systematic means recommended by the manufacturer

What is the purpose of FDC NOTAMs?

To advise of changes in flight data which affect instrument approach procedures (IAP), aeronautical charts, and flight restrictions prior to normal publication.

Which cloud types would indicate convective turbulence?

Towering Cumulus clouds.

(Refer to Area 3.) What is the recommended communications procedure for a landing at Currituck County Airport?

Transmit intentions on 122.9 MHz when 10 miles out and give position reports in the traffic pattern.

Altimeter setting is the value to which the barometric pressure scale of the altimeter is set so the altimeter indicates

True altitude at field elevation

Altimeter setting is the value to which the barometric pressure scale of the altimeter is set so the altimeter is set so the altimeter indicates

True altitude at field elevation

The line from point A to point B of the wind triangle represents

True heading and airspeed

(Refer to Area 5.) The CAUTION box denotes what hazard to aircraft?

Unmarked balloon on cable to 3,008 feet MSL.

Which combination of weather-producing variables would likely result in cumuliform-type clouds, good visibility, and showery rain?

Unstable, moist air and orographic lifting.

(Refer to Area 4.) What hazards to aircraft may exist in restricted areas such as R-5302A?

Unusual, often invisible, hazards such as aerial gunnery or guided missiles.

(Refer to Area 5.) The navigation facility at Dallas-Ft. Worth International (DFW) is a

VOR/DME.

Which would decrease the stability of an air mass?

Warming from below

Refer to Area 3.) Which airport is located at approximately 47°21'N latitude and 101°01'W longitude?

Washburn.

CFR 61.51 (Pilot logbooks) When may instrument flight time be logged?

When a flight is conducted solely by reference to instruments or simulated flight conditions.

Under what condition is indicated altitude the same as true altitude?

When at sea level under standard conditions

CFR 61.3 (Requirements for certificates, ratings, and authorizations) When must a pilot hold an IFR rating?

When operating under IFR or in weather conditions less than VFR minimums.

Under which condition will pressure altitude be equal to true altitude?

When standard atmospheric conditions exist

What is the dotted outlined area northeast of Gila Bend Airport, near Area 3?

Wilderness area.

Advection fog has drifted over a coastal airport during the day. What may tend to dissipate or lift this fog into low stratus clouds?

Wind 15 knots or stronger.

The line from point C to point A of the wind triangle represents

Wind direction and velocity

Under what condition, if any, may pilots fly through a restricted area?

With the controlling agency's authorization.

Refer to Crawford Airport (N38°42.25' W107°38.62'). Is fuel ever available at Crawford Airport?

Yes, 100LL fuel is available for emergency use only.

You are avoiding a thunderstorm that is in your flightpath. You are over 20 miles from the cell however, you are under the anvil of the cell. Is this a hazard?

Yes, hail can be discharged from the anvil.

The wind system associated with a low-pressure area in the Northern Hemisphere is

a cyclone and is caused by Coriolis force.

Every physical process of weather is accompanied by or is the result of

a heat exchange.

During departure, under conditions of suspected low-level wind shear, a sudden decrease in headwind will cause

a loss in airspeed equal to the decrease in wind velocity.

(Refer to Area 6.) Sky Way Airport is

a nonpublic-use airport.

(Refer to Area 1.) Dubey Airport is

a privately owned airport restricted to use.

Ice pellets encountered during flight normally are evidence that

a warm front is about to pass.

What is one purpose of wing flaps? a) To enable the pilot to make steeper approaches to a landing without increasing the airspeed b) To relieve the pilot of maintaining continuous pressure on the controls c) to decrease wing area to vary the lift

a) To enable the pilot to make steeper approaches to a landing without increasing the airspeed

Required landing distances differ at various altitudes and temperatures due to changes in _________.

air density

Air density varies inversely with ________. For example... air is very dense at low ___, and less dense at high ___.

altitude

Things that affect air density (4)

altitude temperature humidity barometric pressure

A situation most conducive to the formation of advection fog is

an air mass moving inland from the coastline during the winter.

While flying cross-country in the Northern Hemisphere, you experience a continuous left crosswind which is associated with a major wind system. This indicates that you

are flying toward an area of generally unfavorable weather conditions.

Distance of the weight from the datum (fixed position on the longitudinal axis of the airplane)

arm

Airborne weather radar is installed to help the crew

avoid severe weather.

One of the most dangerous features of mountain waves is the turbulent areas in and

below rotor clouds.

(Refer to Area 6.) The Class C airspace at Metropolitan Oakland International (OAK) which extends from the surface upward has a ceiling of

both 2,100 feet and 3,000 feet MSL.

point of balance along the airplane's longitudinal axis

center of gravity (CG)

The Low Level Wind Shear Alert System (LLWAS) provides wind data and software process to detect the presence of a

change in wind direction and/or speed within a very short distance above the airport.

Turbulence that is encountered above 15,000 feet AGL not associated with cumuliform cloudiness, including thunderstorms, should be reported as

clear air turbulence.

Provides information at various gross weights, altitudes and temperatures

climb performance chart

With regard to windflow patterns shown on surface analysis charts; when the isobars are

close together, the pressure gradient force is greater and wind velocities are stronger.

Area C on the airport depicted is classified as a

closed runway

To avoid landing at the wrong airport or runway, pilots should

consult airport diagrams and Chart Supplements.

Which would increase the stability of an air mass?

cooling from below

Hail is most likely to be associated with

cumulonimbus clouds

What effect, if any, does high humidity have on aircraft performance? A.Increases performance B.Decreases performance C.No effect on performance

decreases

Which is true regarding actual air temperature and dew point temperature spread? The temperature spread

decreases as the relative humidity increases.

Measurement of the density of the air expressed in terms of altitude

density altitude

Effect of air density on lift and drag (inversely vs directly)

directly

FAA advisory circulars are available to all pilots and are obtained by

downloading them from the FAA website.

What effect does high density altitude, as compared to low density altitude, have on propeller efficiency and why? A.Efficiency is increased due to less friction on propeller blades B.Efficiency is reduced because the propeller exerts less force at high density altitudes than at low density altitudes C.Efficiency is reduced due to the increased force of the propeller in thinner air

efficiency is reduced because the propeller exerts less force at high density altitudes than at low density altitudes

Consists of the airframe, engine and all items of operating equipment permanently installed in the airplane

empty weight

Fog produced by frontal activity is a result of saturation due to

evaporation of precipitation.

Ice pellets encountered during flight are normally evidence that

freezing rain exists at higher altitude.

Weather information is available at the Coeur d'Alene (COE) Airport (Area 2)

from AWOS 3 135.075.

If an emergency situation requires a downwind landing, pilots should expect a faster A.Airspeed at touchdown, a longer ground roll and better control throughout the landing roll B.Groundspeed at touchdown, a longer ground roll, and the likelihood of overshooting the desired touchdown point C.Groundspeed at touchdown, a shorter ground roll, and the likelihood of undershooting the desired touchdown point

groundspeed at touchdown, a longer ground roll, and the likelihood of overshooting the desired touchdown point

The greatest threats to an aircraft operating in the vicinity of thunderstorms are:

hail and turbulence.

Conditions that reduce airplane takeoff and climb performance (3)

high altitude high temperature high humidity

Which combination of atmospheric conditions will reduce aircraft takeoff and climb performance? A.Low temperature, low relative humidity and low density altitude B.High temperature, low relative humidity, low density altitude C.High temperature, high relative humidity, high density altitude

high temperature, high relative humidity, high density altitude

If weight is added to the airplane, it must fly at a (higher/lower?) angle of attack to maintain a given altitude speed.

higher

If the outside air temperature (OAT) at a given altitude is warmer than standard, the density altitude is A.Equal to pressure altitude B.Lower than pressure altitude C.Higher than pressure altitude

higher than pressure altitude

Airplane will require a longer takeoff run on a ___ day. (hot or cold?)

hot

On initial climbout after takeoff and with the autopilot engaged, you encounter icing conditions. In this situation you can expect

ice to accumulate on the underside of the wings due to the higher AOA.

A common location of clear air turbulence is

in an upper trough on the polar side of a jet stream.

Hazardous wind shear is commonly encountered

in areas of temperature inversion and near thunderstorms

As air temperature decreases, density altitude will A.Decrease B.Increase C.Remain the same

increase

GIVEN: Winds at 3,000 feet AGL 30 kts Surface winds Calm While on approach for landing, under clear skies with convective turbulence a few hours after sunrise, one should

increase approach airspeed slightly above normal to avoid stalling.

Which factor would tend to increase the density altitude at a given airport? A.Increase in barometric pressure B.Increase in ambient temperature C.Decrease in relative humidity

increase in ambient temperature

An increase in temperature with an altitude increase

is indication of an inversion.

Which is true regarding a cold front occlusion? The air ahead of the warm front

is warmer than the air behind the overtaking cold front.

What is an important characteristic of wind shear?

it can be present at any level and can exist in both a horizontal and vertical direction

Convective circulation patterns associated with sea breezes are caused by

land absorbing and radiating heat faster than the water.

When flying low over hilly terrain, ridges, or mountain ranges, the greatest potential danger from turbulent air currents will usually be encountered on the

leeward side when flying into the wind.

When flying into a low-pressure area in the Northern Hemisphere, the wind direction and velocity will be from the

left and increasing.

The more moisture in the air, the (more/less?) dense the air.

less

The difference found by subtracting the temperature of a parcel of air theoretically lifted from the surface to 500 millibars and the existing temperature at 500 millibars is called the

lifted index. (negative if lifted & would be warmer i.e. unstable, Positive if lifted & would be colder i.e. stable)

A pilot reporting turbulence that momentarily causes slight, erratic changes in altitude and/or attitude should report it as

light turbulence.

Convective currents are most active on warm summer afternoons when winds are

light.

The jet stream and associated clear air turbulence can sometimes be visually identified in flight by

long streaks of cirrus clouds.

The Devils Lake East MOA (Area 1) is a

military operations area.

When turbulence causes changes in altitude and/or attitude, but aircraft control remains positive, that should be reported as

moderate.

Which situation would most likely result in freezing precipitation? Rain falling from air which has a temperature of

more than 32°F into air having a temperature of 32°F or less.

A strong wind shear can be expected

on the low-pressure side of a jetstream core where the speed at the core is stronger than 110 knots.

There is a high pressure system that is located south of your planned route in the Northern Hemisphere on a west to east cross-country flight. To take advantage of favorable winds, you would plan your route

on the north side of the high pressure area.

The general circulation of air associated with a high-pressure area in the Northern Hemisphere is

outward, downward, and clockwise.

During an approach, the most important and most easily recognized means of being alerted to possible wind shear is monitoring the

power and vertical velocity required to remain on the proper glidepath.

You have planned a cross country flight on a warm spring morning. Your course includes a mountain pass, which is at 11,500 feet MSL. The service ceiling of your airplane is 14,000 feet MSL. After checking the local weather report, you are able to calculate the density altitude of the mountain pass as 14,800 feet MSL. Which of the following is the correct action to take? A. Re-plan your journey to avoid the mountain pass B. Continue as planned since density altitude is only a factor for takeoff C. Continue as planned because mountain thermals will assist your climb.

re-plan your journey to avoid the mountain pass

Flight through a restricted area should not be accomplished unless the pilot has

received prior authorization from the controlling agency.

High density (reduces or increases??) an airplane's overall performance.

reduces

What effect does high density altitude have on aircraft performance? A.Increases engine performance B.Reduces climb performance C.Increases takeoff performance

reduces climb performance

Which is a characteristic of stable air?

restricted visibility, stratiform clouds.

In the Northern Hemisphere, the wind is deflected to the

right by the Coriolis force.

Indicated airspeed for landing is the..... A.Same at all altitudes B.Higher at higher altitudes C.Lower at lower altitudes

same at all altitudes

During the winter months in the middle latitudes, the jet stream shifts toward the

south and speed increases.

The most severe weather conditions, such as destructive winds, heavy hail, and tornadoes, are generally associated with

squall lines.

The formation of either predominantly stratiform or predominantly cumuliform clouds is dependent upon the

stability of the air being lifted.

The conditions most favorable to wave formation over mountainous areas are a layer of

stable air at mountaintop altitude and a wind of at least 20 knots blowing across the ridge.

The conditions necessary for the formation of stratiform clouds are a lifting action and

stable, moist air.

If clouds form as a result of very stable, moist air being forced to ascend a mountain slope, the clouds will be

stratus type with little vertical development and little or no turbulence.

Virga is best described as

streamers of precipitation trailing beneath clouds which evaporates before reaching the ground

What lifts advection fog into low stratus clouds?

surface winds of approximately 15 knots or stronger

(Refer to Area 1.) For information about the parachute jumping at Caddo Mills Airport, refer to

the Airport/Facility Directory section of the Chart Supplement.

Refer to Area 3.) For information about glider operations at Ridgeland Airport, refer to

the Chart Supplement.

When are two-way radio communications required on a flight from Gnoss Airport (DVO) (Area 4) to Livermore Airport (LVK) (Area 5) at an altitude of 3,500 ft. MSL? When entering

the Livermore Airport Class D airspace.

As you approach an airport to land, you observe a convective cloud over the airport with virga below it. This could indicate

the presence of a microburst.

Propellers have less efficiency due to high density because...

there is less air for the propeller to get a grip on

Low-level wind shear may occur when

there is low-level temperature inversion with strong winds above the inversion

Traffic patterns in effect at Lincoln Municipal are

to the left on Runway 14 and Runway 32; to the right on Runway 18 and Runway 35.

Center of gravity computation

total moments/total weight

The conditions necessary for the formation of cumulonimbus clouds are a lifting action and

unstable, moist air.

Which items are included in the empty weight of an aircraft? A.Unusable fuel and undrainable oil B.Only the airframe, powerplant and optional equipment C.Full fuel tanks and engine oil to capacity

unusable fuel and undrainable oil

The presence of standing lenticular altocumulus clouds is a good indication of

very strong turbulence.

(Refer to Area 2.) The flag symbol at Lake Drummond represents a

visual checkpoint used to identify position for initial callup to Norfolk Approach Control.

The flag symbols at Statesboro Bulloch County Airport, Claxton-Evans County Airport, and Ridgeland Airport are

visual checkpoints to identify position for initial callup prior to entering Savannah Class C airspace.

A pilot and two passengers landed on a 2,100 foot east-west gravel strip with an elevation of 1,800 feet. The temperature is warmer than expected and after computing the density altitude it is determined the takeoff distance over a 50 foot obstacle is 1,980 feet. The airplane is 75 pounds under gross weight. What would be the best choice? A.Taking off into the headwind will give extra climb-out time needed B.Try a takeoff without passengers to make sure the climb is adequate C.Wait until the temperature decreases, and recalculate the takeoff performanceFr

wait until temperature decreases, recalculate takeoff performance

On Surface Analysis Charts, widely spaced isobars indicate a

weak pressure gradient.

The strength and location of the jet stream is normally

weaker and farther north in the summer.

Formula for weight and balance

weight x arm = moment

If a temperature inversion is encountered immediately after takeoff or during an approach to a landing, a potential hazard exists due to

wind shear.


Conjuntos de estudio relacionados

AP European History Unit 3 Study Guide

View Set

Chapter 10 Multiple Choice review A&P

View Set

Computer Fundamentals Module 8 - Networking - Quiz

View Set